LSAT 1 – Section 4 – Question 12

You need a full course to see this video. Enroll now and get started in less than a minute.

Target time: 0:52

This is question data from the 7Sage LSAT Scorer. You can score your LSATs, track your results, and analyze your performance with pretty charts and vital statistics - all with a Free Account ← sign up in less than 10 seconds

Question
QuickView
Type Tags Answer
Choices
Curve Question
Difficulty
Psg/Game/S
Difficulty
Explanation
PT1 S4 Q12
+LR
Weaken +Weak
A
59%
163
B
3%
155
C
14%
151
D
24%
154
E
1%
134
142
154
166
+Harder 144.766 +SubsectionEasier
This page shows a recording of a live class. We're working hard to create our standard, concise explanation videos for the questions in this PrepTest. Thank you for your patience!

This is a weakening question, as the question stem asks: Which one of the following, if true, casts the most doubt on the author’s hypothesis?

This is a fairly straight forward correlation-causation argument. A study finds that smokers are more likely to snore, and concludes that smoking can cause snoring. Our job is to weaken this argument. Let’s see what we get:

Correct Answer Choice (A) This provides an explanation for why snoring and smoking would correlate, even when smoking might have no influence on snoring, namely that the two share a third cause, stress.

Answer Choice (B) Ok, but unless we know obesity leads to snoring this does nothing for us.

Answer Choice (C) This is completely compatible with the argument.

Answer Choice (D) Same as C.

Answer Choice (E) Interesting! But this could be true while smoking caused snoring, and does nothing to suggest it doesn’t.

Take PrepTest

Review Results

Leave a Reply